Schwungrad auf halbem Weg durch den Ereignishorizont eines Schwarzen Lochs gegen das Äquivalenzprinzip

Stellen Sie sich vor, ich befinde mich in einer isolierten Rakete von beliebig kleiner Größe und habe ein sich drehendes Schwungrad direkt neben mir. Nehmen wir nun an, meine Rakete passiert den Ereignishorizont / Schwarzschild-Radius eines einfachen Schwarzschild-Schwarzen Lochs.

Nach dem Äquivalenzprinzip sollte ich mich selbst und die Rakete nicht bemerken, die den Ereignishorizont passiert. Da jedoch klassischerweise kein Objekt dem Schwarzen Loch entkommen kann, sobald es den Ereignishorizont passiert, scheint es, als ob das Schwungrad brechen sollte, wenn es den Ereignishorizont passiert, denn für jedes Stück, das in eine Richtung geht, geht das entgegengesetzte Stück davon in die entgegengesetzte Richtung Richtung. Sobald das Schwungrad auf halbem Weg durch den Ereignishorizont ist, kann der Teil des Schwungrads innerhalb des Schwarzen Lochs nicht herauskommen, obwohl es sich drehen muss, so dass es so aussieht, als ob ein Teil des Schwungrads in zwei Hälften geteilt würde .

Wie passt das zum Äquivalenzprinzip?

Mir ist bewusst, dass das Äquivalenzprinzip nur lokal an der Grenze zu immer kleineren Regionen gilt. Mithilfe von Gezeiteneffekten können Sie beispielsweise Regionen mit Schwerkraft und Regionen ohne Schwerkraft unterscheiden. Ich glaube jedoch nicht, dass das ausreicht, um mein Dilemma zu lösen. Wir können davon ausgehen, dass das Schwarze Loch ausreichend groß ist, so dass keine Probleme mit Gezeiteneffekten oder Spaghettifizierungen auftreten. Wir können das Schwarze Loch so groß machen, wie wir wollen, und die Rakete so klein, wie wir wollen, um Gravitationseffekte zweiter Ordnung zu entfernen, und es scheint, als ob mein Paradoxon, dass das Schwungrad den Schwarzschild-Radius kreuzt, immer noch existiert. Liege ich mit dieser Behauptung falsch?

Das ganze Schwungrad fällt in das Schwarze Loch, richtig? Es ist also nicht so, dass eine Hälfte herauskommt, sondern dass eine Hälfte langsamer hineingeht. Es sei denn, Sie bringen ein massives Raketentriebwerk an, in diesem Fall wird das Schwungrad vom Raketentriebwerk auseinandergerissen, nicht vom Schwarzen Loch
Ich denke, die Antworten für physical.stackexchange.com/questions/187917/… decken dies im Wesentlichen ab.
Kommentare sind nicht für längere Diskussionen gedacht; Diese Konversation wurde in den Chat verschoben .

Antworten (2)

Da klassischerweise kein Objekt dem Schwarzen Loch entkommen kann, sobald es den Ereignishorizont passiert, scheint es, als ob das Schwungrad brechen sollte, wenn es den Ereignishorizont passiert, denn für jedes Stück, das in eine Richtung geht, geht das entgegengesetzte Stück davon in die entgegengesetzte Richtung.

Diese Analyse ist falsch. Der Ereignishorizont ist eine lichtähnliche Fläche. In einem lokalen Inertialsystem bewegt es sich bei c nach außen. Es stimmt also, dass es ein antipodisches Stück gibt, das in die andere Richtung geht, aber es spielt keine Rolle. Das Antipodenstück bewegt sich im lokalen Inertialsystem langsamer als c. Der Horizont bewegt sich also schneller und das antipodische Stück kann unmöglich den Horizont zurückkreuzen. Das Schwungrad dreht sich ohne Unterbrechung weiter und ohne Gefahr, den Horizont rückwärts zu überqueren.

Ich kann dieser Antwort nicht folgen. AIUI, die Frage fragt nach der Kontinuitätsgleichung in räumlich ausgedehnten Systemen, wenn sie den Ereignishorizont überschreiten. (Hier ist das System ein Schwungrad und Kontinuitätsgesetz Massentransport.) Da sich der Ereignishorizont "nach außen bewegt C ", der Zeitraum, in dem sich ein Teil des Systems (links) innerhalb des Horizonts befindet und ein Teil (rechts) außerhalb zeitliche Ausdehnung hat. Während dieser Zeit muss so viel der erhaltenen Menge nach links wie nach rechts gehen, aber das ist unmöglich, weil nichts kreuzt der Horizont nach rechts. Wird das Kontinuitätsgesetz verletzt? Wenn ja, warum können wir die Verletzung nicht messen?
@JacobManaker sagte: "So viel der konservierten Menge muss nach links wie nach rechts gehen, aber das ist unmöglich". Dies ist keine korrekte Aussage. Unmöglich ist, dass die Erhaltungsgröße den Ereignishorizont von innen nach außen durchquert. Damit dies geschieht, muss die Erhaltungsgröße nicht nur nach rechts gehen, sondern dass sie schneller als c (im lokalen Inertialsystem) nach rechts geht.
Ja genau! Sie haben beschrieben, was physikalisch passieren muss. Aber das Kontinuitätsgesetz zeigt etwas anderes an, damit wir nicht auf der linken Seite des Horizonts eine "Anhäufung" sehen. (Das würde einer Unwucht des Schwungrades entsprechen, denke ich.) Warum/wie versagt hier das Kontinuitätsgesetz?
@JacobManaker Das Kontinuitätsgesetz gibt nichts anderes an. Nicht sicher, warum Sie denken, dass es so ist. Vielleicht sollten Sie dazu eine separate Frage stellen, bei der Sie etwas Platz zum Erklären haben
Ich denke, es hilft sehr, darauf hinzuweisen, dass der Ereignishorizont eine lichtähnliche Oberfläche ist. In der Minkowski-Raumzeit passiert ständig lichtähnliche Oberflächen. Ich werde in den nächsten Tagen noch etwas darüber nachdenken.
@Dale: Beim Schreiben meiner Frage habe ich meine Schwierigkeit herausgefunden. Vielen Dank für Ihre Geduld mit diesen vagen Kommentaren.
@JacobManaker Ich habe diese Erfahrung selbst gemacht. Manchmal besteht die meiste (oder sogar die gesamte) Arbeit, um eine Antwort zu erhalten, darin, einfach eine gute Frage zu stellen. Klingt so, als ob deine Frage so gut gewesen sein muss
@MaximalIdeal: Ich versuche gerade herauszufinden, wie ich die Antwort tatsächlich mit statischer Geometrie formulieren kann. Es stellt sich heraus, dass es sich um einen sehr erfundenen Fall handelt, bei dem Sie es bemerken können.

Dies ist keine direkte Antwort, sondern eine Untersuchung einer analogen Situation, die mir geholfen hat, Dales Antwort zu verstehen. Ich poste es hier, falls jemand anderes die Diskussion anschaulich findet. (Aber Sie sollten Daves Antwort trotzdem positiv bewerten!)

Diese Frage gehört zu einer allgemeineren Klasse von Phänomenen: einem räumlich ausgedehnten System, in dem sich eine Erhaltungsgröße in einer Schleife bewegt. Beispiele beinhalten:

  • ein rotierendes Schwungrad (Erhaltungsgröße: Masse),
  • ein Stromkreis (Erhaltungsgröße: elektrische Ladung) und
  • Flüssigkeit, die durch eine Rohrschlaufe gepumpt wird (Erhaltungsgröße: Masse, Verwirbelung, Schwebeteilchen – alles, was von der Flüssigkeit transportiert wird).

Es ist sehr einfach, die Situation zu analogisieren, in der die Erhaltungsgröße Elektrizität ist und gegen den Uhrzeigersinn fließt. Das schöne konzeptionelle Merkmal dieses analogen Systems ist, dass es uns eine klare Unterteilung in die Schwarzloch-internen und -externen Teile geben kann.

Betrachten Sie dazu eine willkürliche Teilung des Systems in zwei Abschnitte. Ein Abschnitt L liegt auf der linken Seite; der andere Abschnitt R auf der rechten Seite. Wir können davon ausgehen, dass der Teilungspunkt bei Position liegt X sowohl oben als auch unten, wo es nichts Komplizierteres als einen Draht kreuzt. Wichtig ist jedoch, dass wir nicht davon ausgehen können, dass diese Teilung zeitinvariant ist. Der Ereignishorizont wird unser System mit Lichtgeschwindigkeit passieren; X muss damit reisen.

Zu jedem gegebenen Zeitpunkt können wir dann unser System durch zwei Größen und vier (vorzeichenbehaftete) Flüsse von rechts nach links beschreiben:

  • Q L , die Gesamtladung auf der linken Seite;
  • Q R , die Gesamtladung rechts;
  • ich T , F , die Strömung entlang der Spitze bei X (halten X Konstante);
  • ich T , B , der Pseudostrom, der die Ladungen vorübergehend fixiert und die Grenze entlang der Spitze bewegt;
  • ich B , F , die Strömung entlang der Unterseite an X ; Und
  • ich B , B , der untere Pseudostrom.

Nehmen Sie das vorübergehend an X ist konstant. Dann ich T , B = ich B , B = 0 . Durch die Kontinuitätsgleichung haben wir

D Q L D T = ich T , F + ich B , F
Aber wir können nicht unbegrenzte Ladung auf der linken Seite akkumulieren lassen, also müssen wir im langfristigen Gleichgewicht haben
(1) ich T , F = ich B , F
Tatsächlich können wir bereits davon ausgehen, dass das System dieses Gleichgewicht erreicht hat. Seit ich T , F Und ich B , F werden durch Halten definiert X konstant, (1) muss immer gelten. Da der Strom gegen den Uhrzeigersinn fließt, ist jede Seite von (1) positiv.

Lassen Sie nun das System in ein schwarzes Loch fallen (links) und wählen Sie X immer mit dem Ereignishorizont abschließen. Nach dem Äquivalenzprinzip sollte dieser Zeitraum, wenn unser System ausreichend klein ist, "nichts Besonderes" sein.

Wir können kombinieren ich T , F Und ich T , B um die Gesamtladungen zu erhalten, die oben und unten in das Schwarze Loch fallen (jeweils):

ich T = ich T , F + ich T , B ich B = ich B , F + ich B , B
Aber nichts kann einen Ereignishorizont verlassen, also müssen wir es haben ich T 0 Und ich B 0 .

Dies ist also der Kern des "Paradoxons": Unsere Intuition wird durch Situationen geformt, in denen D X D T ist klein, wenn nicht 0 . In diesem Fall,

ich B ich B , F < 0
Wenn wir in das schwarze Loch fallen, ein großes und positives ich B , B muss stattdessen dominieren.

Aber seit D X D T = C , ein großes, positives ich B , B ist nicht schwer zu arrangieren.